oslo90066
Thanks Received: 0
Vinny Gambini
Vinny Gambini
 
Posts: 12
Joined: June 28th, 2009
 
 
trophy
First Responder
 

Q12 - Politician: It is wrong for the government

by oslo90066 Wed Dec 02, 2009 12:52 am

I understand logically why the right answer is A for this question, but the first time I did this question, I eliminated A and E because I didn´t think ¨not right¨ meant the same thing as ¨wrong¨, which is in the stimulus. I am wondering if someone could explain this aspect of the answer choice.
User avatar
 
noah
Thanks Received: 1192
Atticus Finch
Atticus Finch
 
Posts: 1541
Joined: February 11th, 2009
 
This post thanked 2 times.
 
 

Re: Q12 - Politician: It is wrong for the government

by noah Wed Dec 02, 2009 2:31 pm

What a tricky question!

To answer your specific question, it actually doesn't matter whether "not right" and "wrong" are interchangeable. I agree that it seems a bit fishy, since perhaps there's something in the middle -- a neutral act, right? As is typical with the LSAT, that issue has been side-stepped. The correct answer is correct because we can combine the statements as such:

Restricting liberty --> Wrong (if avoiding caveat of harm) [and thus, Not Wrong --> Not Restrict Liberty]
Not cause harm --> Wrong to restrict liberty [this is a tricky one, except acts like "unless", and this is the more important statement to understand here]
Publish --> Liberty
Offend --> Not cause harm

Therefore, we can do a replacement chain of sorts:
publishing (a liberty) that only offends --> publishing (a liberty) that does not cause harm --> wrong to restrict publishing.

Thus, it's not right to restrict publishing that is only offensive, as (A) states.

To hit it over the head a bit more: we know that if something is only offending, it's wrong to restrict it. We can confidently say that something that is wrong is not right. And, we've avoided deciding whether something that is not right is wrong! Analogously, if we know that there is oil and water, we cannot say that not water = oil, but we can say that water is not oil, and we can say that oil is not water.

I know that's a bit crazy-sounding, so tell me if you have some more questions about that.

As for the other answers:

(B) is very tempting! It sure seems like it isn't wrong for the government to restrict harm-causing stuff. But, the stimulus only tells us that it's PERHAPS not wrong. It still might be. If you look up, we never could infer that harm --> right to restrict. We only know that if not harm --> wrong to restrict

(C) is about what sorts of government acts are offensive. We don't know anything about that.

(D) is out of scope. The right and wrong facts are all about the government restricting things, not us common folk actually doing stuff.

(E) is tempting. But, it might be OK for the government to restrict publications that don't cause serious harm since they might still cause harm!
 
claudia.minoiu
Thanks Received: 0
Vinny Gambini
Vinny Gambini
 
Posts: 6
Joined: April 09th, 2013
 
 
 

Re: Q12 - Politician: It is wrong for the government

by claudia.minoiu Mon Sep 02, 2013 11:24 pm

B and E are traps - they are both STRONGER than the stimulus. This tip I share with everyone helped me not lose many points on LSAT: on inference questions you never ever ever EVER want to select an answer that is stronger than the stimulus.

B) is wrong because the stimulus says PERHAPS (see how LSAT introduces these words that change everything??)
E) is wrong because the stimulus says CAUSE HARM. Not SERIOUS harm.
User avatar
 
WaltGrace1983
Thanks Received: 207
Atticus Finch
Atticus Finch
 
Posts: 837
Joined: March 30th, 2013
 
This post thanked 1 time.
 
trophy
Most Thanked
trophy
Most Thankful
trophy
First Responder
 

Re: Q12 - Politician: It is wrong for the government

by WaltGrace1983 Mon Apr 14, 2014 1:48 pm

Alright guys, I found a simpler solution to this problem. I decided not to think about it like an inference question but rather I tried to just look at the ones in which the necessary/sufficient conditions align, a time-saving trick that principle questions often test. What I mean is that in principle questions you will often see something like "If Sally walked the dog, John walked the cat" and the incorrect choices will say something like (_______ → Sally walked the dog) or (John walked the cat → ________), obviously being wrong because they mistake the necessary/sufficient conditions. Let's break this down like a friggin' enzyme :geek:

Let's think about what we know:
    (1) It is wrong to restrict liberty
    (2) If failing to restrict liberty would NOT allow individuals to cause harm, then it is wrong to restrict that liberty
    (3) To publish is a liberty
    (4) To offend is not to cause harm


So we have these four facts and they are very much conditional so we can put them into a logical form:
    (1) Restrict liberty → Wrong
    (2) ~Harm → Wrong
    (3) Publish → Liberty
    (4) Offend → ~Harm


I'll add a caveat to this because (1) and (2) are not both sufficient in themselves to bring about (Wrong). What I mean is that simply restricting liberty does not mean that restricting that liberty undoubtedly wrong. Why? Because what if something restricts liberty, but also would cause harm if that liberty were not restricted? This wouldn't be wrong! The stimulus says so! Look at the stimulus again and read it carefully if you don't know what I mean. Therefore, (1) and (2) have to be combined together! In order to be wrong, it must both (restrict liberty) and (~harm).

    (Restrict Liberty) AND (~Harm) → Wrong
    ~Wrong → (~Restrict Liberty) OR (Harm)


This is really all we need. Now comes that little time-saving trick I was talking about earlier. Look at the conclusions of every answer choice. You don't need to bother with the premises yet but just look at the conclusions. Do you see something funny about them in comparison to the diagrams we have above?


    (A) _______ → ~Right
    (B) _______ → ~Wrong
    (C) _______ → Offensive
    (D) _______ → ~Wrong
    (E) _______ → ~Right

      We can automatically eliminate (B), (C), and (D)! Why? Because the conclusions of those answer choices do not match any possible conclusions of our stimulus's diagram. We only know (_______ → Wrong) or (~Wrong → _______). Anything else we simply cannot infer! Let's get rid of (B), (C), and (D).


Now what about (A) and (E)? We don't know anything about (~Right)! So why are these okay? They are okay because if we can conclude from a set of premises that something is WRONG then we can similarly conclude from those same premises that something is NOT RIGHT. If Wrong → ~Right; if Right → ~Wrong. This is just the plain definition of the word. So let's look at (A) and (E)

    (E) is...

      Restrict Publication & ~Serious Harm → ~Right


    (A) is...

      Restrict Publication & Offensive → ~Right




(E) oversteps the boundaries of the stimulus. Just because something doesn't cause serious harm does not mean that it does not cause harm at all. A paper cut is not serious harm; it is however harm nonetheless.

I also would like to mention something neat about (A). It says that it is only offensive. This is really cool because, once again, we aren't 100% certain that restricting liberty and not causing harm is 100% sufficient to bring about wrong. Maybe an action could restrict liberty and not cause harm but it would instead cause something else that is clearly wrong (I cannot think of anything off of the top of my head that would not involve causing harm but still...). (A) probably doesn't absolutely NEED the "only" offensive part, but it is a rock solid answer choice and shows how well this question was written.

Hope that helps!
Last edited by WaltGrace1983 on Sat Apr 26, 2014 12:27 pm, edited 1 time in total.
 
christine.defenbaugh
Thanks Received: 585
Atticus Finch
Atticus Finch
 
Posts: 536
Joined: May 17th, 2013
 
This post thanked 2 times.
 
 

Re: Q12 - Politician: It is wrong for the government

by christine.defenbaugh Thu Apr 24, 2014 3:27 pm

I've always LOVED this method of solving this type of question, WaltGrace1983!

I think I may be able to help you simplify it even further. Don't split that first sentence into two separate conditionals. The conditional keyword "except perhaps" functions exactly the same way that "unless" functions. So, we can think of that first sentence as giving us a rule that says:

If you don't fall into the exception clause, then gov't restriction on liberty IS WRONG.

We have no earthly idea what kinds of things would guarantee something being right, or offensive. All we've got is a rule that will prove wrongness in certain situations.

That swiftly and powerfully eliminates (B), (C), and (D), just as you note! And we are safe with the remaining answers that conclude something is "not right", because if our nifty rule gets triggered, we'll know that something is wrong - which surely means it ain't right!

Now, we have to assess whether (A) or (E) triggers our nifty rule. One of them has got fall clearly outside the exception clause - that's the only way we'll know that it's wrong. And that exception clause has something to do with harm. For sanity's sake, I simplify it this way:

    If a thing is not harmful --> wrong to restrict.

(A) talks about something that is "only offensive" - well, what do we know about offensive stuff? We know it doesn't, by itself, cause harm. So, really, we're talking about a definitively not harmful thing. And the only way for the government to avoid the conclusion of wrongness is to invoke the spectre of harm! This ain't going to cut it to get the gov't off the hook - it's still wrong!

And what about (E)? This talks about something that's not seriously harmful. Well, that's nice, but what if it's a little harmful, in a non-serious way? Would it fall into the exception clause about harm? I don't know! Maybe?! We can't conclude that this thing is or isn't wrong! If it causes a little harm, maybe it's still okay for the gov't to restrict it.

Simplified, these answers look like this, side by side:
(A) Non-harmful thing --> wrong to restrict
(E) Not seriously harmful thing --> wrong to restrict

I 10000% endorse the method of eliminating answers that come up with results we have no way of knowing ('not wrong' and 'offensive') as a first attack! Just be careful not to overcomplicate the conditional you're working with!

Keep up the truly excellent work!
User avatar
 
WaltGrace1983
Thanks Received: 207
Atticus Finch
Atticus Finch
 
Posts: 837
Joined: March 30th, 2013
 
 
trophy
Most Thanked
trophy
Most Thankful
trophy
First Responder
 

Re: Q12 - Politician: It is wrong for the government

by WaltGrace1983 Sat Apr 26, 2014 12:26 pm

"Except perhaps" is also equivalent to plain ol' "except" right, Christine?
 
christine.defenbaugh
Thanks Received: 585
Atticus Finch
Atticus Finch
 
Posts: 536
Joined: May 17th, 2013
 
This post thanked 1 time.
 
 

Re: Q12 - Politician: It is wrong for the government

by christine.defenbaugh Sat May 03, 2014 11:06 pm

WaltGrace1983 Wrote:"Except perhaps" is also equivalent to plain ol' "except" right, Christine?


Yes indeed! "Except", "except perhaps", "unless", and "until" all function the same way!
User avatar
 
Mab6q
Thanks Received: 31
Atticus Finch
Atticus Finch
 
Posts: 290
Joined: June 30th, 2013
 
 
 

Re: Q12 - Politician: It is wrong for the government

by Mab6q Mon Jan 19, 2015 9:40 pm

WaltGrace1983 Wrote:Alright guys, I found a simpler solution to this problem. I decided not to think about it like an inference question but rather I tried to just look at the ones in which the necessary/sufficient conditions align, a time-saving trick that principle questions often test. What I mean is that in principle questions you will often see something like "If Sally walked the dog, John walked the cat" and the incorrect choices will say something like (_______ → Sally walked the dog) or (John walked the cat → ________), obviously being wrong because they mistake the necessary/sufficient conditions. Let's break this down like a friggin' enzyme :geek:

Let's think about what we know:
    (1) It is wrong to restrict liberty
    (2) If failing to restrict liberty would NOT allow individuals to cause harm, then it is wrong to restrict that liberty
    (3) To publish is a liberty
    (4) To offend is not to cause harm


So we have these four facts and they are very much conditional so we can put them into a logical form:
    (1) Restrict liberty → Wrong
    (2) ~Harm → Wrong
    (3) Publish → Liberty
    (4) Offend → ~Harm


I'll add a caveat to this because (1) and (2) are not both sufficient in themselves to bring about (Wrong). What I mean is that simply restricting liberty does not mean that restricting that liberty undoubtedly wrong. Why? Because what if something restricts liberty, but also would cause harm if that liberty were not restricted? This wouldn't be wrong! The stimulus says so! Look at the stimulus again and read it carefully if you don't know what I mean. Therefore, (1) and (2) have to be combined together! In order to be wrong, it must both (restrict liberty) and (~harm).

    (Restrict Liberty) AND (~Harm) → Wrong
    ~Wrong → (~Restrict Liberty) OR (Harm)


This is really all we need. Now comes that little time-saving trick I was talking about earlier. Look at the conclusions of every answer choice. You don't need to bother with the premises yet but just look at the conclusions. Do you see something funny about them in comparison to the diagrams we have above?


    (A) _______ → ~Right
    (B) _______ → ~Wrong
    (C) _______ → Offensive
    (D) _______ → ~Wrong
    (E) _______ → ~Right

      We can automatically eliminate (B), (C), and (D)! Why? Because the conclusions of those answer choices do not match any possible conclusions of our stimulus's diagram. We only know (_______ → Wrong) or (~Wrong → _______). Anything else we simply cannot infer! Let's get rid of (B), (C), and (D).


Now what about (A) and (E)? We don't know anything about (~Right)! So why are these okay? They are okay because if we can conclude from a set of premises that something is WRONG then we can similarly conclude from those same premises that something is NOT RIGHT. If Wrong → ~Right; if Right → ~Wrong. This is just the plain definition of the word. So let's look at (A) and (E)

    (E) is...

      Restrict Publication & ~Serious Harm → ~Right


    (A) is...

      Restrict Publication & Offensive → ~Right




(E) oversteps the boundaries of the stimulus. Just because something doesn't cause serious harm does not mean that it does not cause harm at all. A paper cut is not serious harm; it is however harm nonetheless.

I also would like to mention something neat about (A). It says that it is only offensive. This is really cool because, once again, we aren't 100% certain that restricting liberty and not causing harm is 100% sufficient to bring about wrong. Maybe an action could restrict liberty and not cause harm but it would instead cause something else that is clearly wrong (I cannot think of anything off of the top of my head that would not involve causing harm but still...). (A) probably doesn't absolutely NEED the "only" offensive part, but it is a rock solid answer choice and shows how well this question was written.

Hope that helps!


What an LSAT baller. One of the best explanations I've read. Thanks.
"Just keep swimming"
User avatar
 
tommywallach
Thanks Received: 468
Atticus Finch
Atticus Finch
 
Posts: 1041
Joined: August 11th, 2009
 
 
 

Re: Q12 - Politician: It is wrong for the government

by tommywallach Wed Jan 21, 2015 5:14 pm

Walt is a beast. : )
Tommy Wallach
Manhattan LSAT Instructor
twallach@manhattanprep.com
Image
 
contropositive
Thanks Received: 1
Atticus Finch
Atticus Finch
 
Posts: 105
Joined: February 01st, 2015
 
 
 

Re: Q12 - Politician: It is wrong for the government

by contropositive Wed Feb 04, 2015 10:52 pm

I don't even understand why people are writing this out in logical format. You are wasting time by doing that because you can easily get this by process of elimination, which is helpful for must be true question types.

B) incorrect negation of first sentence. Eliminate

C) it's offensive for government? Nice mixing up words in the stimulus, but don't fall for it. Eliminate

D) could be true but not must be true. In fact, we know what is wrong and right for government to do when it comes to restrictions individual liberty but do we know what's wrong and right for individuals to do?

E) serious harm? This is like the correct answer choice (A) except it has the phrase "serious harm" the stimulus talks about harm not serious harm. Thus, A is better answer choice.


Now why go through writing out the logic when you could just be picky at it. When I read a must be true I am very picky about everything in each answer choice, even the right one but often it turns out that 4 of them are SO wrong compared the the one that is kinda weird but the right answer choice.
 
blairped
Thanks Received: 0
Vinny Gambini
Vinny Gambini
 
Posts: 13
Joined: March 30th, 2015
 
 
 

Re: Q12 - Politician: It is wrong for the government

by blairped Mon Dec 07, 2015 8:40 am

This is how I ended up with (A) after POE like how Walt did above:

1. RL-->W
2. ~H-->W
3. P-->L
4. O-->~H
------------------
= RL (P-->L) & ~H (O-->~H) ----> W


DECONSTRUCT (A)!

= RL (P-->L) & ~H (O-->~H) ----> W
= to restrict the publication of literature that is only offensive ----> It is not right for the government


RL= to restrict the publication of LITERATURE (=LIBERTY)
~H= that is only offensive (AND HENCE DOES CAUSE HARM)
W= It is not right for the government
 
zcxlwj
Thanks Received: 0
Vinny Gambini
Vinny Gambini
 
Posts: 8
Joined: October 05th, 2015
 
 
 

Re: PT 39, S2, Q12 Politician: It is wrong for the government to

by zcxlwj Sun Jun 12, 2016 7:16 pm

noah Wrote: P.S. The fact that (B) seemed to restate a premise makes it suspicious.


Hello - I have a question about general test taking strategy.

Noah mentioned an answer choice that appears to be restating a premise may be suspicious. I recall reading elsewhere on the forum that, for an Inference question, being too similar to a statement in the stimulus is not a good reason not to select an answer choice.

Could you someone please help clarify? Also, is there any difference between "infer" and "properly infer"? Thanks.
User avatar
 
ManhattanPrepLSAT1
Thanks Received: 1909
Atticus Finch
Atticus Finch
 
Posts: 2851
Joined: October 07th, 2009
 
 
 

Re: Q12 - Politician: It is wrong for the government

by ManhattanPrepLSAT1 Wed Jun 22, 2016 4:17 pm

zcxlwj Wrote:Hello - I have a question about general test taking strategy.

Noah mentioned an answer choice that appears to be restating a premise may be suspicious. I recall reading elsewhere on the forum that, for an Inference question, being too similar to a statement in the stimulus is not a good reason not to select an answer choice.

Could you someone please help clarify? Also, is there any difference between "infer" and "properly infer"? Thanks.


Hey zcxlwj, I think he meant suspicious as in worth attention. Is it restating the information accurately, or possibly twisted in some way? The answer to that will determine whether it follows from the statements above. So what you read earlier is correct.

Also, there is no difference between "infer" and "properly infer," but do pay attention to the difference between "infer" and "most supported." That issue has been a challenge for students historically.
 
zcxlwj
Thanks Received: 0
Vinny Gambini
Vinny Gambini
 
Posts: 8
Joined: October 05th, 2015
 
 
 

Re: Q12 - Politician: It is wrong for the government

by zcxlwj Thu Jun 23, 2016 10:07 am

Thanks for clarifying, Matt. Very helpful!

Since you brought it up, would you mind adding some color around the difference between "infer" and "most supported," or point me in the right direction? :)
User avatar
 
ManhattanPrepLSAT1
Thanks Received: 1909
Atticus Finch
Atticus Finch
 
Posts: 2851
Joined: October 07th, 2009
 
 
 

Re: Q12 - Politician: It is wrong for the government

by ManhattanPrepLSAT1 Fri Jun 24, 2016 5:14 pm

Sure, it's simply a matter of degree. "Infer" means must be true, while "most supported" means more support than anything else. "Most Supported" questions have a lower bar of proof than "Must be True" questions.

Hope that answers your question!
 
zcxlwj
Thanks Received: 0
Vinny Gambini
Vinny Gambini
 
Posts: 8
Joined: October 05th, 2015
 
 
 

Re: Q12 - Politician: It is wrong for the government

by zcxlwj Sat Jun 25, 2016 11:37 am

Yes, super helpful. Thanks again, Matt!
 
aaronwfrank
Thanks Received: 2
Vinny Gambini
Vinny Gambini
 
Posts: 23
Joined: August 24th, 2016
 
 
 

Re: Q12 - Politician: It is wrong for the government

by aaronwfrank Mon Oct 10, 2016 2:23 pm

mattsherman Wrote:Sure, it's simply a matter of degree. "Infer" means must be true, while "most supported" means more support than anything else. "Most Supported" questions have a lower bar of proof than "Must be True" questions.

Hope that answers your question!



Hey Matt. I'm curious about the diagramming on this question. How do you deal with the "except" in the first sentence? I've seen the sufficient switched around with the necessary like that once or twice and it's confusing to know when to switch them around. I actually don't even know what the conditions are here.

All I have is

Restrict Liberty-->Wrong

Restricting Liberty-->Not Wrong -->Cases Failing To Do So Would Cause Harm (I'VE NO IDEA HOW TO COMBINE THE FIRST AND SECOND PARTS OF THE SENTENCE)

Publishing-->Liberty

Offend-->Not Cause Harm
 
HannaR18
Thanks Received: 0
Vinny Gambini
Vinny Gambini
 
Posts: 1
Joined: March 12th, 2020
 
 
 

Re: Q12 - Politician: It is wrong for the government

by HannaR18 Thu Mar 12, 2020 4:59 am

Hi,

Although I appreciate the brilliant and efficient method explained in the earlier posts, I really wanted to solve this problem by using formal logic to see if I truly understand how to translate words to conditional reasoning:

*1ST STATEMENT*

(Restrict L -> Wrong) UNLESS (failing to Restrict L -> Cause Harm)
= 'equals'
~ (RL -> W) -> (L -> CH)

Here because of UNLESS, I negated the sufficient condition, and also "failing to restrict" is same as couldn't/didn't restrict hence crossing out both negative terms results in simply "L" meaning liberty as it is without any restrictions.

Now, I am not sure how to apply negation to this equation (RL -> W). But I thought if we negate A-> B, that must mean when there is A, there is no B, hence A-> ~B. So, if we apply this to the above equation,

we can write: ~(RL -> W) = (RL -> ~W)

plug that into the original statement, we get (RL -> ~W) -> (L -> CH)

In English, "if restricting liberty is not wrong, then liberty leads to causing harm." Sounds about right.


*2ND STATEMENT*

Publishing -> Liberty
Offend -> ~CH



*INFERENCE*

So, we end up with three equations:

(RL -> ~W) -> (L -> CH)
P -> L
O -> ~CH *contrapositive CH -> ~O

Let's chain them up by connecting P to L and CH to ~O from the contrapositive:

(RL -> ~W) -> (P -> L -> CH -> ~O)

Since LSAT normally asks for the furthest inference, we end up with

(RL -> ~W) -> (P -> ~O)

Take the contrapositive by flipping and negating both conditions, we get

~(P -> ~O) -> ~(RL -> ~W)

apply the negation sign to both conditions

ULTIMATELY, (P -> O) -> (RL -> W)

Since according to the stimulus, "publishing is a liberty,"

"if publishing is only offensive, then it is wrong to restrict that liberty of publishing something."


which is exactly same as the answer choice A which states,

"It is not right (=wrong) for the government to restrict (a liberty of) the publication that is only offensive."


Yay! Now, I can get some sleep lol! For those of you who are losing sleep over this question because you couldn't put the stimulus in pure formal logic and couldn't find any posts that do exactly so, I hope my post helps you find peace of mind.

Cheers,


Hanna
 
Laura Damone
Thanks Received: 94
Atticus Finch
Atticus Finch
 
Posts: 468
Joined: February 17th, 2011
 
 
 

Re: Q12 - Politician: It is wrong for the government

by Laura Damone Sat Mar 28, 2020 4:45 pm

First, please excuse the delayed response. This one must have slipped through the cracks amidst all the craziness of the past few weeks.

Second, Nice work, Hannah!

That was clean, but it was definitely complicated.

A simpler way to integrate formal logic into the approach for this question is this:

Restrict Liberty + NOT prevent harm --> Wrong 2 Restrict

Publish --> Liberty

Offend --> -Harm
------------------------------------------------------

Restricting the publishing of something that's only offensive is wrong


That first conditional that you diagrammed is definitely accurate, but it's overly complex. The basic statement is "A is wrong unless -A leads to B." So, "A + -B --> A is wrong."

Hope this helps!
Laura Damone
LSAT Content & Curriculum Lead | Manhattan Prep
 
Emmeline Ndongue
Thanks Received: 0
Jackie Chiles
Jackie Chiles
 
Posts: 36
Joined: September 12th, 2017
 
 
 

Re: Q12 - Politician: It is wrong for the government

by Emmeline Ndongue Wed Oct 27, 2021 9:43 am

I'd like to share my 2 cents on (A) and (E)
Hardest part of the question: except=if not, perhaps=maybe (like 50% chance)

The 1st sentence actually means:
If it's [not necessarily] wrong for the gov. to restrict the liberty of individual, perhaps in those cases when fail to do so (restrict the liberty of individual) would allow individual to cause harm.
OR the contrapositive:
If perhaps in those cases when fail to restrict the liberty of individual would [not necessarily] allow individual to cause harm, it's wrong for the gov. to do so (restrict the liberty of individual).

The 2nd sentence:
first part: to publish sth is a liberty. Noted this doesn't mean Publish->liberty (to publish anything is a liberty). We don't know publishing what kind of stuff is considered a liberty.
latter part: to offend is not to cause harm. This actually means:
To offend is [not necessarily] to cause harm. (Offend->~cause harm is WRONG! it would mean anything that offends won't cause harm, ie offend is sufficient to guarantee not causing harm)

(A) "publication of literature that's only offensive". This tells us that "publication of literature" doesn't cause harm (or is harmful), because if sth is only offensive then it can't be both offensive and harmful. However, whether it is considered a liberty is unknown. This is one problem of the correct answer choice. Another problem is that, even if you assume that it's a liberty, it doesn't really trigger the conditional relationship in the 1st sentence, because of the word "perhaps", the conditional relationship could or couldn't happen. Even if you are lenient on that, it doesn't match the phrasing of the sufficient condition of the contrapositive, as it's "when FAIL TO RESTRICT THE LIBERTY OF INDIVIDUAL would ALLOW INDIVIDUAL to cause harm". The capital letters show the difference in phrasing. Last but not least, can you say "not right" equal "wrong"? of course not! There are things that people find hard to be right nor wrong. There are those vague, in-between things. Unless you define what's wrong clearly, which the stimulus didn't.

(E) "publication of literature that doesn't cause serious harm". Sth that doesn't cause serious harm can still cause harm, so we know that "publication of literature" might cause harm, but again we don't know if "publication of literature" is a liberty, like mentioned above, it doesn't really fit the conditional relationship in the 1st sentence, so it's incorrect.

You have to be loose on the language and the conditional relationship in order to quickly, efficiently pick (A). I don't think this is a strictly designed question.